*NURSING > HESI MED SURG > Med_Surg 2021 EXAM 250 Q&A with Rationales. NURSING nursing 42 | Already Graded A. (All)

Med_Surg 2021 EXAM 250 Q&A with Rationales. NURSING nursing 42 | Already Graded A.

Document Content and Description Below

1) A female client with a nasogastric tube attached to low suction states that she is nauseated. The nurse assesses that there has been no drainage through the nasogastric tube in the last 2 hours. Wh... ich action should the nurse take first? A. Irrigate the nasogastric tube with sterile normal saline. B. Reposition the client on her side. C. Advance the nasogastric tube 5 cm. D. Administer an intravenous antiemetic as prescribed. B. The priority is to determined if the tube is functioning correctly, which would relieve the client's nausea. The least invasive intervention is to reposition the client (B), should be attempted first, followed by (A & C) if these are unsuccessful then (D). 2) When assigning clients on a medical-surgical floor to a RN and a LPN, it is best for the charge nurse to assign which client to the LPN? A. A child with bacterial meningitis with recent seizures. B. An older adult client with pneumonia and viral meningitis. C. A female client in isolation wiht meningococcal meningitis. D. A male client 1 day post-op after drainage of a brain abscess. B. Is the most stable. A, C, D have an increased risk for elevated ICP. 3) Which description of symptoms is characteristic of a client with diagnosed with trigeminal neuralgia (tic douloureux)? A. Tinnitus, vertigo, and hearing difficulties. B. Sudden, stabbing, severe pain over the lip and chin. C. Unilateral facial weakness and paralysis. D. Difficulty in talking, chewing, and swallowing. B. Trigeminal neuralgia is characterized by paroxysms of pain, similar to an electric shock, in the area innervated by one or more branches of the trigeminal nerve. A. Characteristic of Meniere's C. Characteristic of Bell palsey D. Characteristic of disorders of the hypoglossal (12th cranial nerve) 4) Which abnormal lab finding indicates that a client with diabetes needs further evaluation for diabetic nephropathy? A. Hypokalemia B. Microalbuminauria C. Elevated serum lipids D. Ketonuria B. Microalbuminuria is the earliest sign of nephropathy and indicates the need for follow-up evaluation. Hyperkalemia (A) is associated with end stage renal disease caused by diabetic nephropathy. (C) may be elevated in end stage renal disease. (D) may signal the onset of DKA. 5) An older male client comes to the geriatric screening clinic complaining of pain in his left calf. The nurse notices a reddened area on the calf of his right leg that is warm to touch and the nurse suspects that the client may have thrombophlebitis. Which addition assessment is most important for the nurse to perform? A. Measure calf circumference. B. Auscultate the client's breath sounds. C. Observe for ecchymosis and petechiae. D. Obtain the client's blood pressure. B. Since the client may have a pulmonary embolus secondary to the thrombophlebitis. A. Would support the nurses assessment. C. Least helpful since bruising is not associated with thrombophlebitis. D. Less important then auscultation. 6) The nurse know that a client taking diuretics must be assessed for the development of hypokalemia, and that hypokalemia will create changes in the client's normal ECG tracing. Which ECG change would be an expected finding in the client with hypokalemia? A. Tall, spiked T waves B. A prolonged QT interval C. A widening QRS complex D. Presence of a U wave D. A U wave is a positive deflection following the T wave and is often present with hypokalemia. A, B, C indicate hyperkalemia. 7) An older client is admitted with a diagnosis of bacterial pneumonia. The nurse's assessment of the client will most likely reveal which S/SX? A. Leukocytosis and febrile. B. Polycythemia and crackles. C. Pharyngitis and sputum production. D. Confusion and tachycardia. D. The onset of pneumonia is the older may be signaled by general deterioration, confusion, increased heart rate or increased respiratory rate. (A, B, C) are often absent in the older with bacterial pneumonia. 8) The nurse observes ventricular fibrillation on telemetry and upon entering the clients bathroom finds the client unconscious on the floor. What intervention should the nurse implement first? A. Administer an antidysrhythmic medication. B. Start cardiopulmonary resuscitation. C. Defibrillate the client at 200 joules. D. Assess the client's pulse oximetry. B. Ventricular fibrillation is a life-threatening dysrhythmia and CPR should be started immediately. A & C are appropriate but B is the priority. D does not address the seriousness of the situation. 9) An older female client with dementia is transferred from a long term care unit to an acute care unit. The client's children express concern that their mother's confusion is worsening. How should the nurse respond? A. "It is to be expected that older people will experience progressive confusion." B. "Confusion in an older person often follows relocation to new surroundings." C. "The dementia is progressing rapidly, but we will do everything we can to keep your mother safe." D. "The acute care staff is not as experienced as the long-term care staff at dealing with dementia." B. Relocation often results in confusion among older clients and is stressful to clients of all ages. (A) is an inaccurate stereotype. (C) is most likely false there are many factors that cause increased temporary confusion. (D) may be true but does not offer the family a sense of security about the care. 10) The nurse plans to help an 18-year-old developmentally disabled female client ambulate on the first postoperative day. When the nurse tells her it is time to get out of bed, the client becomes angry and yells at the nurse. "Get out of here! I'll get up when I'm ready." Which response should the nurse provide? A. "Your healthcare provider has prescribed ambulation on the first postoperative day." B. "You must ambulate to avoid serious complications that are much more painful." C. "I know how you feel; you're angry about having to do this, but it is required." D. "I'll be back in 30 minutes to help you get out of bed and walk around the room." D. Returning in 30 minutes provides a cooling off period, is firm, direct, nonthreatening, and avoids argument with the client. B is threatening. C. assumes what the client is feeling. A. avoids the nurse's responsibility to ambulate the client. 11) The nurse is performing hourly neurological check for a client with a head injury. Which new assessment finding warrants the most immediate intervention by the nurse? A. A unilateral pupil that is dilated and nonreactive to light. B. Client cries out when awakened by a verbal stimulus. C. Client demonstrates a loss of memory to the events leading up to the injury. D. Onset of nausea, headache, and vertigo. A. Any changes in pupil size and reactivity is an indication of increasing ICP and should be reported immediately. (B) is normal for being awakened. (C & D) are common manifestations of head injury and less of an immediacy than (A). 12) A male client with arterial peripheral vascular disease (PVD) complains of pain in his feet. Which instruction should the nurse give to the UPA to quickly relieve the client's pain? A. Help the client to dangle his legs. B. Apply compression stockings. C. Assist with passive leg exercises. D. Ambulate three times daily. A. A client who has arterial PVD may benefit from a dependent position which can be achieved by dangling by improving blood flow and relieving pain. (B) is indicated for venous insufficiency and (C) is indicated for bed rest. (D) is indicated to facilitate collateral circulation and may improve long term complaints of pain. 13) A 58-year-old client, who has no health problems, asks the nurse about taking the pneumococcal vaccine (Pneumovax). Which statement give by the nurse would offer the client accurate information about this vaccine? A. "The vaccine is given annually before the flue season to those over 50 years of age." B. "The immunization is administered once to older adults or persons with a history of chronic illness." C. "The vaccine is for all ages and is given primarily to those person traveling overseas to infected areas." D. "The vaccine will prevent the occurrence of pneumococcal pneumonia for up to 5 years." B. It is usually recommended that persons over 65 years of age and those with a history of chronic illness should receive the vaccine once in a lifetime. (A) the influenza vaccine is given annually. (C) travel is not the main rationale for the vaccine. (D) The vaccine is usually given once in a lifetime. 14) A client with hypertension has been receiving ramipril (Altace) 5 mg PO daily for 2 weeks and is scheduled to receive a dose at 0900. At 0830 the client's blood pressure is 120/70. Which action should the nurse take? A. Administer the dose as prescribed. B. Hold the dose and contact the healthcare provider. C. Hold the dose and recheck the blood pressure in 1 hour. D. Check the healthcare provider's prescription to clarify the dose. A. The BP is WNL and indicates that the medication is working. (B & C) would be indicated if the BP was low (systole below 100). (D) is not required because the dose is within manufacture's recommendations. 15) The nurse know that normal lab values expected for an adult may vary in an older client. Which data would the nurse expect to find when reviewing laboratory values of an 80-year-old man who is in good overall health. A. Complet blood count reveals increased WBC and decreased RBC counts. B. Chemistries reveal an increased serum bilirubin with slightly increased liver enzymes. C. Urinalysis reveals slight protein in the urine and bacteriuria with pyuria. D. Serum electrolytes reveal a decreased sodium level with an increased potassium level. C. In older adults the protein found in urine is slightly risen as a result of kidney changes or subclinical UTIs and the client frequently experiences asymptomatic bacteriuria and pyuria as a result of incomplete bladder emptying. (A, B, D) are not normal findings. 16) The nurse is completing an admission inter for a client with Parkinson disease. Which question will provide addition information about manifestations the client is likely to experience? A. "Have you ever experienced and paralysis of your arms or legs?" B. " Do you have frequent blackout spells?" C. "Have you ever been 'frozen' in one spot, unable to move?" D. "Do you have headaches, especially ones with throbbing pain?" C. Parkinson clients frequently experience difficulty in initiating, maintaining, and performing motor activities. They may even experience being rooted, unable to move. (A, B, D) Does not typically occur in Parkinson. 17) During the change of shift report, the charge nurse reviews the infusions being received by the clients on the oncology unit. The client receiving which infusion should be seen first? C. Has the highest risk for respiratory depression and therefor should be seen first. (A) Risk of hypotension. (B) Lowest risk. (D) Risk of nephrotoxicity and phlebitis. 18) The home health nurse is assessing a male client being treated for Parkinson disease with levodopa-carbidopa (Sinemet). The nurse observes that he does not demonstrate any apparent emotions when speaking and rarely blinks. Which intervention should the nurse implement? A. Perform a complete cranial nerve assessment. B. Instruct the client that he may be experiencing medication toxicity. C. Document the presence of these assessment findings. D. Advise the client to seek immediate medical evaluation. C. A mask-like expression and infrequent blinking are common clinical features of Parkinsonism. The nurse should document the findings. (A & D) are not necessary. Signs of toxicity (B) are dyskinesia, hallucinations, and psychosis. 19) A client is placed on a mechanical ventilator following a cerebral hemorrhage, and vecuronium bromide (Norcuron) 0.04 mg/kg every 12 hours IV is prescribed. What is the priority nursing diagnosis for this client? A. Impaired communication related to paralysis of skeletal muscles. B. Hight risk or infection related to increased ICP. C. Potential for injury related to impaired lung expansion. D. Social isolation related to inability to communicate. A. To increase the client's tolerance of the endotracheal intubation and/or mechanical ventilation, a skeletal-muscle relaxant such as vecuronium is usually prescribed. (A) is a serious outcome because the client cannot communicate his/her needs. (D) is not as much of a priority. (B) infection is not related to ICP. (C) is incorrect because the ventilator will ensure that the lungs are expanded. 20) The nurse is reviewing the routine medications taken by a client with chronic angle closure glaucoma. Which medication prescription should the nurse question? A. An antianginal with a therapeutic effect of vasodilation. B. An anticholinergic with a side effect of pupillary dilation. C. An antihistamine with a side effect of sedation. D. A corticosteroid with a side effect of hyperglycemia. B. Clients with angle closure glaucoma should not take medications that dilate the pupil (B) because this can precipitate acute and severely increased intraocular pressure. (A, C, D) do not cause increased intraocular pressure, which is the primary concern. 21) What is the correct location for the placement of the hand for manual chest compressions during CPR on the adult client. A. Just above the xiphoid process on the upper third of the sternum. B. Below the xiphoid process midway between the sternum and the umbilicus. C. Just about the xiphoid process on the lower third of the sternum. D. Below the xiphoid process midway between the sternum and the first rib. Ans: C 22) Twelve hours after chest tube insertion for hemothorax, the nurse notes that the client's drainage has decreased from 50 ml/hr to 5 ml/hr. What is the best inital action for the nurse to take? A. Document this expected decrease in drainage. B. Clamp the chest tube while assessing for air leaks. C. Milk the tube to remove any excessive blood clot build up. D. Assess for kinks or dependent loops in the tubing. D. The least invasive action should be performed to assess the decrease in drainage. (A) is completed after assessing for and problems causing the decreased drainage. (B) is no longer protocol because the increased pressure may be harmful for the client. (C) is an appropriate nursing action after the tube has been assessed for kinks or dependent loops. 23) A 25-year-old client was admitted yesterday after a motor vehicle collision. Neurodiagnostic studies showed a basal skull fracture in the middle fossa. Assessment on admission revealed both halo and Battle signs. Which new symptom indicates that the client is likely to be experiencing a common life-threatening complication associated with basal skull fracture? A. Bilateral jugular vein distention. B. Oral temperature of 102 degrees F. C. Intermittent focal motor seizures. D. Intractable pain in the cervical region. B. Increased temp indicates meningitis. (C & D) these symptoms may be exhibited but are not life threatening. (A) JVD is not a typical complication of basal skull fractures. 24) Seconal 0.1 gram PRN at bedtime is prescribed for rest. The scored tablets are labeled grain 1.5 per tablet. How many tablets should the nurse plan to administer? A. 1/2 tablet B. 1 tablet C. 1 1/2 tablet D. 2 tablets B. 15 gr = 1 g, 0.1 x 15 = 1.5 grains 25) Which content about self-care should the nurse include in the teaching plan of a client who has genital herpes? (Select all that apply.) A. Encourage annual physical and Pap smear. B. Take antiviral medication as prescribed. C. Use condoms to avoid transmission to others. D. Warm sitz baths may relieve itching. E. Use Nystatin suppositories to control itching. F. Douche with weak vinegar solutions to decrease itching. A,B,C,D. (E) is specific for Candida infections and (F) is used to treat Trichomonas. 26) A client with chronic asthma is admitted to postanesthesia complaining of pain at level 8 of 10, with a BP of 124/78, pulse of 88 beats/min, and respirations of 20 breaths/min. The postanesthesia recovery prescription is, "Morphine 2 to 4 mg IV push while in recovery for pain level over 5." What intervention should the nurse implement? A. Give the medication as prescribed to decrease the client's pain. B. Call the anesthesia provider for a different medication for pain. C. Use nonpharmacologic techniques before giving the medication. D. Reassess pain level in 30 Minutes and medicate if it remains elevated. B. Call for a different medication because morphine and meperidine (Demerol) have histamine- releasing narcotics and should be avoided when a client has asthma. (A) puts the client at risk for asthma attack. (C & D) disregard the clients prescription and pain relief. 27) During report, the nurse learns that a client with tumor lysis syndrome is receiving an IV infusion containing insulin. Which assessment should the nurse complete first? A. Review the client's history for diabetes mellitus. B. Observe the extremity distal to the IV site. C. Monitor the client's serum potassium and blood glucose. D. Evaluate the client's oxygen saturation and breath sounds. C. The client with tumor lysis syndrome may experience hyperkalemia, therefor it is important to monitor serum potassium and blood glucose levels. (A, B, D) are not as priority. 28) During assessment of a client in the intensive care unit, the nurse notes that the client's breath sounds are clear upon auscultation, but jugular vein distention and muffled heart sounds are present. Which intervention should the nurse implement? A. Prepare the client for a pericardial tap. B. Administer intravenous furosemide (Lasix). C. Assist the client to cough and deep breathe. D. Instruct the client to restrict oral fluid intake. A. The client is exhibiting symptoms of cardiac tamponade that results in reduced cardiac output. Treatment is pericardial tap. (B) is not a treatment. (C) is not priority. (D) Fluids are frequently increased but this is not as priority as (A). 29) In assessing an older client with dementia for sundowning syndrome, what assessment technique is best for the nurse to use? A. Observe for tiredness at the end of the day. B. Perform a neurologic exam and mental status exam. C. Monitor for medication side effects. D. Assess for decreased gross motor movement. A. Sundowning syndrome is a pattern of agitated behavior in the evening, believed to be associated with tiredness at the end of the day combined with fewer orienting stimuli, such as activities and interactions. (B, C, & D) with not provide information about this syndrome. 30) Which condition should the nurse anticipate as a potential problem in a female client with a neurogenic bladder? A. Stress incontinence. B. Infection. C. Painless, gross hematuria. D. Peritonitis. B. Infection is the major complication resulting from stasis of urine and subsequent catheterization. (A) is the involuntary loss of urine through an intact urethra as a result of suddenly increased pressure. (C) is the most common symptom of bladder cancer. (D) is the most common and serious complication of peritoneal dialysis. 31) The nurse is interviewing a client who is taking interferon-alfa-2a (Roferon-A) and ribavirin (Virazole) combination therapy for hepatitis C. The client reports experiencing overwhelming feelings of depression. What action should the nurse implement first? A. Recommend mental health counseling. B. Review the medications actions and interactions. C. Assess for the client's daily activity level. D. Provide information regarding a support group. B. Alpha-interferon and ribavirin combination therapy can cause severe depression. (A, B, C) may be implemented after physiological aspect of the situation are assessed. 32) The nurse is assessing a 75-year-old male client for symptoms of hyperglycemia. Which symptom of hyperglycemia is an older adult most likely to exhibit? A. Polyuria B. Polydipsia C. Weight loss D. Infection D. S/Sx of hyperglycemia in older adults may include fatigue, infection, and neuropathy (such as sensory changes). (A, B, C) are classic symptoms and may be absent in the older adult. 33) A client who is receiving an ACE inhibitor for hypertension calls the clinic and reports the recent onset of a cough to the nurse. What action should the nurse implement? A. Advise the client to come to the clinic immediately for further assessment. B. Instruct the client to discontinue use of the drug, and make an appointment at the clinic. C. Suggest that the client lear to accept the cough as a side effect to a necessary prescription. D. Encourage the client to keep taking the drug until seen by the HCP. D. Cough is a common s/e of ACE inhibitors and is not an indication to discontinue the medication. (A) immediate evaluation is not needed. (B) an antihypertensive should not be stopped abruptly. (C) is demeaning since the cough may be disruptive to the client and other medications may produce results without the s/e. 34) The nurse is observing an unlicensed assistive personnel (UPA) who is performing morning care for a bedfast client with Huntington disease. Which care measure is most important for the nurse to supervise? A. Oral care B. Bathing C. Foot care D. Catheter care A. A client with Huntington disease experiences problems with motor skills such as swallowing and is at high risk for aspiration. (B, C, D) do not pose life-threatening consequences. 35) A client with alcohol-related liver disease is admitted to the unit. Which prescription should the nurse question as possibly inappropriate for the client? A. Vitamin K1 (AquaMEPHYTON) 5 mg IM daily B. High-calorie, low-sodium diet C. Fluid restriction to 1500 ml/day D. Pentobarbital (Nembutal sodium) 50 mg at bedtime for rest D. Sedatives such as Nembutal are contraindicated for clients with liver damage and can have dangerous consequences. (A) is often prescribed since normal clotting mechanism is damaged. (B) is needed to restore energy. (C) Fluids are restricted to decrease ascites which often accompanies cirrhosis, particularly in later stages of the disease. 36) A client diagnosed with chronic kidney disease (CDK) 2 years ago is regularly treated at a community hemodialysis facility. In assessing the client before his scheduled dialysis treatment, which electrolyte imbalance should the nurse anticipate? A. Hypophosphatemia B. Hypocalcemia C. Hyponatremia D. Hypokalemia B. Hypocalcemia develops in CKD due to chronic hyperphosphatemia not (A). (C & D) incorrect you would find hypernatremia and hyperkalemia 37) Debilitating anginal pain can be decreased in some clients by the administration of beta- blocking agents such as nadolol (Corgard). Which client requires the nurse to use extreme caution when administering Corgard? A. A 56-year-old air traffic controller who had bypass surgery 2 years ago. B. A 47-year-old kindergarten teacher diagnosed with asthma 40 years ago C. A 52-year-old unemployed stock broker who refuses treatment for alcoholism D. A 60-year-old retired librarian who takes a diuretic daily for hypertension. B. asthma must be carefully monitored because beta blockers because it can induce cardiogenic shock and reduce bronchodilation efforts. (A & D) this medication is indicated and (C) it is not contraindicated. 38) A male client who has never smoked but has had COPD for the past 5 years is now being assessed for cancer of the lung. The nurse knows that he is most likely to develop which type of lung cancer? A. Adenocarcinoma B. Oat-cell carcinoma C. Malignant melanoma D. Squamous-cell carcinoma A. is the only lung cancer not related to cigarette smoking related to lung scarring and fibrosis from preexisting pulmonary diseases such as TB and COPD. (B& D) are related to smoking. (C) is a skin cancer 39) The nurse is caring for a client with a chest tube to water seal drainage that was inserted 10 days ago because of a ruptured bullae and pneumothorax. Which finding should the nurse report to the healthcare provider before the chest tube is removed? A. Tidal of water in the water seal chamber B. Bilateral muffled breath sounds at bases C. Temperature of 101 degrees F D. Absence of chest tube drainage for 2 days. A. Tidal in the water seal chamber should be reported to the HPC to show that the chest tube is working properly. (B) may indicate hypoventilation from the chest tube and usually improves when the tube is removed. (C) indicates infection (D) is an expected finding. 40) A central venous catheter has been inserted via a jugular vein and a radiography has confirmed placement of the catheter. A prescription has been received for stat medication but IV fluids have not yet been started. What action should the nurse take prior to administering the prescribed medication? A. Assess for signs of jugular vein distention. B. Obtain the needed intravenous solution. C. Administer a bolus of normal saline solution. D. Flush the line with heparinized saline. C. A medication can be administered central line without IV fluids, flush with normal saline to remove heparin that may counteract with the medication. (B) is used following the medication and a second saline bolus. (A) will not impact the the med administration and is not a priority. (B) Administration of the stat medication is more of a priority than (B). 41) The nurse assesses a postoperative client. Oxygen is being administered at 2 L/min and a saline lock is in place. Assessment shows cool, pale, moist skin. The client is very restless and has scant urine in the urinary drainage bag. What intervention should the nurse implement first. A. Measure urine specific gravity. B. Obtain IV fluids for infusion protocol. C. Prepare for insertion of a central venous catheter. D. Auscultate the client's breath sounds. B. The client is at risk for hypovolemic shock and is exhibiting early signs. Start IV to restore tissue perfusion. (A, C, D) are all important but less of a priority. 42) A client diagnosed with angina pectoris complains of chest pain while ambulating in the hallway. Which action should the nurse implement first? A. Support the client to a sitting position. B. Ask the client to walk slowly back to the room. C. Administer a sublingual nitroglycerin tablet. D. Provide oxygen via nasal cannula. A. Assist in safely repositioning and then administer (C & D). Then the client can be escorted back to the room via wheelchair or stretcher (B). 43) A 55-year-old male client is admitted to the coronary care unit having suffered an acute myocardial infarction (MI). Within 24 hours of the occurrence, the nurse can expect to find which systemic sign? A. Elevated serum amylase level B. Elevated CM-MB level C. Prolonged prothrombin time (PT) D. Elevated serum BUN and creatinine B. Tissue damage in the myocardium causes the release of cardiac enzymes into the blood system. An elevated CM-MB is a recognized indicator of an MI. It peaks 12 - 24 hours and returns to normal within 48 - 78 hours. (A) would indicate pancreatitis or a gastric disorder. (D) Although an elevated BUN might be related to an acute MI it is usually associated with dehydration, high protein intake or gastrointestinal bleeding and creatine levels indicate renal damage. (C) Indicates effective anticoagulation therapy. 44) The nurse is assessing a client who presents with jaundice. Which assessment finding is the most significant indication that further follow up is needed? A. Urine specific gravity of 1.03 with a urine output of 500 ml in 8 hours B. Frothy, tea-colored urine C. Clay-colored stools and complaints of pruritus D. Serum amylase and lipase levels that are twice their normal levels D. Obstructive cholelithiasis and alcoholism are the two major causes of pancreatitis, and an elevated serum amylase and lipase indicate pancreatic injury. (A) is a normal finding. (B & C) are expected findings for jaundice. 45) A client with cirrhosis states that his disease was cause by a blood transfusion. What information should the nurse obtain first to provide effective client teaching? A. The year the blood transfusion was received B. The amount of alcohol the client drinks C. How long the client has had cirrhosis D. The client's normal coping mechanisms A. The nurse should first verify the clients explanation (A) since it may be accurate due to prior to 1990 blood was not screened for Hep C and hep C can cause cirrhosis. Not all cirrhosis is caused is caused by alcoholism (B) (C & D) provide useful but less relevant information. 46) What is the correct procedure for performing an ophthalmoscopic examination on a client's right eye? A. Instruct the client to look at the examiner's nose and not move his/her eyes during the exam. B. Set ophthalmoscope on the plus 2 to 3 lens and hold it in front of the examiner's right eye. C. From a distance of 8 to 12 inches and slightly to the side, shine the light into the client's pupil. D. For optimum visualization, keep the ophthalmoscope at least 3 inches for the client's eye C. The client should focus on a distant object in order to promote pupil dilation. The ophthalmoscope should be set on the 0 lens to begin (creates no correction) and should be held in front of the examiner's left eye when examining the client's right eye and kept 1" from the client's eye for optimum visualization. (A, B, D) are incorrect procedures. 47) The nurse witnesses a baseball player receive a blunt trauma to the back of the head with a softball. What assessment data should the nurse collect immediately? A. Reactivity of deep tendon reflexes, comparing upper to lower extremities. B. Vital signs readings, excluding blood pressure if need equipment is unavailable. C. Memory of events that occurred before and after the blow to the head. D. Ability to spontaneously open the eyes before any tactile stimuli are given. D. The LOC should be immediately established immediately after the head injury has occurred. Spontaneous eye opening (D) is a simple measure of LOC. (A) is not the best indicator of LOC. (B) is important but not the best indicator of LOC. (C) can be assessed after LOC has been established by assessing eye opening. 48) A 43-year-old homeless, malnourished female client with a history of alcoholism is transferred to the ICU. She is placed on telemetry, and the rhythm strip shown is obtained. The nurse palpates a heart rate of 160 beats/min, and the client's blood pressure is 90/54. Based on these finding, which IV medication should the nurse administer? A. Amiodarone (Cordarone) B. Magnesium sulfate C. Lidocaine (Xylocaine) D. Procainamide (Pronestyl) B. Because the client has chronic alcoholism, she is likely to have hypomagnesium. (B) is the recommended drug for torsades de pointes (AHA, 2005), which is a form of polymorphic ventricular tachycardia (VT), usually associated with a prolonged QT interval that occurs with hypomagnesemia. (A and D) increase the QT interval, which can cause the torsades to worsen. (C) is the antiarrhythmic of choice in most cases of drug-induced monomorphic VT, not torsades. 49) The nurse is caring for a critically ill client with cirrhosis of the liver who has a nasogastric tube draining bright red blood. The nurse notes that the client's serum hemoglobin and hematocrit are decreased. What additional change in lab data should the nurse expect? A. Increased serum albumin B. Decreased serum creatinine C. Decreased serum ammonia D. Increased liver function tests C. The breakdown of glutamine in the intestine and the increased activity of colonic bacteria from the digestion of proteins increases the ammonia levels in the clients with advanced liver disease, so removal of blood, a protein source, from the intestines results in reduced ammonia. (A, B, D) will not be significantly impacted by the removal of blood. 50) A family member was taught to suction a client's tracheostomy prior to the client's discharge from the hospital. Which observation by the nurse indicates that the family member is capable of correctly performing the suctioning technique? A. Turns on the continuous wall suction to -190 mm Hg B. Inserts the catheter until resistance or coughing occurs C. Withdraws the catheter while maintaining suctioning D. Re-clears the tracheostomy after suctioning the mouth B. indicates correct technique for performing suctioning. Suction pressure should be between -80 and -120 (A). The catheter should be withdrawn 1-2 cm at a time with intermittent suction (C). (D) introduces pathogens. 51) The nurse is planning the care for a client who is admitted with the syndrome of inappropriate antidiuretic hormone secretion (SIADH). Which interventions should the nurse include in this client's plan of care? (Select all that apply.) A. Salt-free diet B. Quiet environment C. Deep tendon reflex assessments D. Neurologic checks E. Daily weights F. Unrestricted intake of free water B, C, D, E. SIADH results in water retention and dilutional hyponatremia, which causes neurologic change when serum sodium levels are less than 115 mEq/L. The nurse should maintain a quiet environment (B) to prevent overstimulation that can lead to periods of disorientation, assess deep tendon reflexes (C) and neurologic checks (D) to monitor for neurologic deterioration. Daily weights (E) should be monitored to assess for fluid overload: 1 kg weight gain equals 1 L of fluid retention, which further dilutes serum sodium levels. (A and F) contribute to dilutional hyponatremia. 52) levodopa (Sinemet) Parkinsons Disease lessen tremors increases amount of levodopa to CNS (dopamine to the brain) s/s toxicity=dyskinesia, hallucinations, psychosis 53)Aldosteronism lab =decreased serum level of potassium hypokalemia hypertension 54)seconel sleep aide 15g=1g 55)Laryngectomy cuff should be inflated only prior to feeding 56)tumor lysis syndrome hyperkalemia may occur =requires insulin to reduce serum potassium = monitor serium potassium and blood glucose levels 57) older adults protein found in urine slightly rises as a result of kidney change or UTI w/asymptomatic bacteriuria and pyuria as a result of incomplete bladder emptying 58) chest tube decreased drainage =assess for kinks or dependant loops -do not clamp off 59)angina pectoris when walking=1.assist to seated position; 2. sublingual nitroglycerin; 3.oxygen 4. wheelchair to room 60) SIADH inappropriate antidiuretic hormone secreation water retention & dilutional hyponatremia POC=quiet enviroment, deep tendon reflex assessment, neurologic checks, daily weights 1kg=1L 61) methotrexate (Mexate) immunosuppressant can cause bone marrow depression rheumatoid arthritis lab=hemaglobin decrease =adverse side effect 62)pentobarbital (Neubatal sodium) contriandicated with liver damage 63)cirrhosis Vitamin K1 (AquaMephyton) high calorie, low sodium diet sodium restriction w/ edema fluids restricted to decrease ascites late stage = ascites 64)TPN total protein nutrition only regular insulin is adm. IV return containing NPH 65)kidney stone strain all urine most important encourage urine 66)blunt trama to back of head LOC assessment most important 67)cancer reduce fats increase fruits, vegetables and fiber ie bran flakes, skim milk, orange slices 68)telemetry ventrical fibrillation = life threatening start CPR 69)COPD contributing factor=smoking 70)osteoporosis most common cause of fractured hip= reduced calcium in bones result of hormonal changes during perimenaopausal years 71)esophagogastromy esophageal cancer risk for infection = meticulious oral care should be provided several times a day prior to surgery 72)diverticulitits hard ridgid abdomen & elevated WBC = peritonitis = medical emergency should be reported to PCP immediately s/s left lower quadrant pain; elevated temperature; refusing to eat; nausea 73)lactulose (Cephulac) reduce blood ammonia by excreation of ammonia by stools 2 -3 soft stools per day 74)IV's infusions potential problems morphine, continous epidural = respiratory depression magnesium continous infusion = hypotension vancomycin intermittent infusion = nephrotoxicity & phlebitits 75)pancreatitis serium amylase & lipase 2 to 5 times higher than normal hypercalcemia 40 ~ 75% = positive trousseau sign = carpal spasm severe boring pain 76)hypercalcemia positive trousseau sign = carpal spasm 77)neurogenic bladder infection - from stasis of urine and subsuquent catheterization 78) CKD chronic kidney disease prior to hemodialysis lab= hypocalcemia due to hyperphosphatemia, hyperkalemic & hypernatremic 79) gangrene necrosis/tissue death priority prevent infection 80)olpthalmoscopic exam from a distance of 8-12 inches and slightly to the side, shine the light in the clients pupil; client should focus on distant object to promote dialation, olpthalmoscopic set at 0 lens to begin, should be held in front of the examiners left eye when examining clients right eye and kept within 1 inch of clients eye for optimum visualation 81) hypokalemia patients on diuretics will change patients normal ECG = U wave is positive deflection following the T wave often present in hypokalemia 82) ECG U wave is positive deflectionfollowing the T wave often present in hypokalemia tall spiked T wave, prolonged QT intervial, widening QRS complex are all signs of hyperkalemia 83)Hyperkalemia ECG=tall spiked T wave, prolonged QT intervial, widening QRS complex are all signs of hyperkalemia; tumor lysis syndrome 84)ACE inhibitor cough os a common side effect hypertension do not stop abruptly (rebound hypertension may occur) 85)permanent pacemaker changes in pulse rate in rythem may indicate pacer failure dizziness may be due to decreased heart rate leading to decreased cardiac output; should carry a card in wallet with type and serial number of pacemaker; report redness and tenderness - s/s infection 86)sundowning agitated behavior in the evening observe for tiredness at the end of the day 87)Pap smear should be continued through menapause to test for vaginal and cervical cancer 88)NG Tube no drainage in 2 hours client nausated = reposition client on side 89)hypomagnesemia chronic alcholic ie HR 160 BP 90/54 give IV magnesium sulphate prolonged QT intervial 90) Magnesium Sulphate hypomagnesemia reccomended for torsadesde pointes a form of polymorphic ventrical tackycardia associated with a prolonged QT intervial that occurs with hypomagnesemia 91) older adults stooped posture results in upper torso becomming center of gravity 92)Cushing Syndrome results from hypersecreation glucocorticoids in the adrenal cortex often develope diabetes mellitus - monitor serum glucose levels generialized edema low calorie, low carbohydrate, low sodium diet 93)jaundice serium amylase & lipase 2 times higher than normal indicate pancreatic injury frothy tea colored urine clay colored stools complaints of puritis 94) PVD peripheral vascular disease help client dangle legs 95) digitalis (Lanoxin) digoxin cardiac glycoside can build up toxic levels s/s anexoria, nausea, vomiting, diarrhea, headache, fatigue 96)Hepatitis B health care providers should have Hep B vaccine; transmitted by fecal/oral contamination 97)nadolol (Corgard) beta blocker dibilatating anginal pain bypass surgery patients use with diuretic for hypertension use extreme caution with respiratory problems (asthma) and congestive heart failure 98)CPR just above the xiphoid process on the lower third of the sternum 99)trigeminal neuralgia (5th crainal nerve) sudden stabbing severe pain over the lip and chin 100)meniere syndrome (8th crainal nerve) tinnitus, vertigo, eharing difficulties 101)Bell palsy (7th crainal nerve) unilateral facial weakness and paralysis 102)Hypoglossal (12th crainal nerve) difficulty chewing, talking and swallowing 103)TSS Staphlococcus aures produce a toxin that can enter the blood stream through vaginal mucosa. wash hands before and change tampon frequently 4-6 hours 104) small bowel obstruction peritonis w/ Temperature of 102 notify HCP immediately abdominal cramping 105) vecuronium bromide (Norcuron) skeletal muscle relaxatant ND: impaired communication R/T paralysis of skeletal muscles 106)neuro function altered neuro function = 107)Amniocentesis Surgical puncture to remove fluid from the sac around the embryo. 108)Tonsillitis Inflammation of lymph tissue in the throat. 109)-ptosis Prolapse 110)Ischemia Blood is held back from an area. 111)Necr/o Death 112)Acromegaly Enlargement of extremities after puberty due to pituitary gland problem. 113)Otalgia Pain in the ear. 114)Chronic Continuing over a long period of time. 115)Arteriole Small artery. 116)-scope Instrument to visually examine. 117)Cystocele Hernia of the urinary bladder. 118)Malignant myeloma Tumor of bone marrow. 119)Myelogram X-ray record of the spinal cord. 120)-cocci Berry shaped bacteria. 121)-graph Instrument to record. 122)-oid Resembling 123)Leukocyte Eosinophil is a (an) 124)Laryngectomy Removal of the voice box. 125)Angioplasty Surgical repair of blood vessel. 126)Lymphocyte A blood cell that produces antibodies. 127)Hypertrophy Excessive devolopment. 128)-therapy Treatment. 129)-stomy Surgical creation of a permanent opening to the outside of the body. 130)Catabolism The process by which food is burned to realease energy. 154) The nurse assesses a patient with shortness of breath for evidence of long-standing hypoxemia by inspecting: A. Chest excursion B. Spinal curvatures C. The respiratory pattern D. The fingernail and its base D. The fingernail and its base Clubbing, a sign of long-standing hypoxemia, is evidenced by an increase in the angle between the base of the nail and the fingernail to 180 degrees or more, usually accompanied by an increase in the depth, bulk, and sponginess of the end of the finger. 155) The nurse is caring for a patient with COPD and pneumonia who has an order for arterial blood gases to be drawn. Which of the following is the minimum length of time the nurse should plan to hold pressure on the puncture site? A. 2 minutes B. 5 minutes C. 10 minutes D. 15 minutes B. 5 minutes Following obtaining an arterial blood gas, the nurse should hold pressure on the puncture site for 5 minutes by the clock to be sure that bleeding has stopped. An artery is an elastic vessel under higher pressure than veins, and significant blood loss or hematoma formation could occur if the time is insufficient. 156) The nurse notices clear nasal drainage in a patient newly admitted with facial trauma, including a nasal fracture. The nurse should: A. test the drainage for the presence of glucose. B. suction the nose to maintain airway clearance. C. document the findings and continue monitoring. D. apply a drip pad and reassure the patient this is normal. A. test the drainage for the presence of glucose. Clear nasal drainage suggests leakage of cerebrospinal fluid (CSF). The drainage should be tested for the presence of glucose, which would indicate the presence of CSF. 157) When caring for a patient who is 3 hours postoperative laryngectomy, the nurse's highest priority assessment would be: A. Airway patency B. Patient comfort C. Incisional drainage D. Blood pressure and heart rate A. Airway patency Remember ABCs with prioritization. Airway patency is always the highest priority and is essential for a patient undergoing surgery surrounding the upper respiratory system. 158) When initially teaching a patient the supraglottic swallow following a radical neck dissection, with which of the following foods should the nurse begin? A. Cola B. Applesauce C. French fries D. White grape juice A. ColaWhen learning the supraglottic swallow, it may be helpful to start with carbonated beverages because the effervescence provides clues about the liquid's position. Thin, watery fluids should be avoided because they are difficult to swallow and increase the risk of aspiration. Nonpourable pureed foods, such as applesauce, would decrease the risk of aspiration, but carbonated beverages are the better choice to start with. 159) The nurse is caring for a patient admitted to the hospital with pneumonia. Upon assessment, the nurse notes a temperature of 101.4° F, a productive cough with yellow sputum and a respiratory rate of 20. Which of the following nursing diagnosis is most appropriate based upon this assessment? A. Hyperthermia related to infectious illness B. Ineffective thermoregulation related to chilling C. Ineffective breathing pattern related to pneumonia D. Ineffective airway clearance related to thick secretions A. Hyperthermia related to infectious illness Because the patient has spiked a temperature and has a diagnosis of pneumonia, the logical nursing diagnosis is hyperthermia related to infectious illness. There is no evidence of a chill, and her breathing pattern is within normal limits at 20 breaths per minute. There is no evidence of ineffective airway clearance from the information given because the patient is expectorating sputum. 160) Which of the following physical assessment findings in a patient with pneumonia best supports the nursing diagnosis of ineffective airway clearance? A. Oxygen saturation of 85% B. Respiratory rate of 28 C. Presence of greenish sputum D. Basilar crackles D. Basilar crackles The presence of adventitious breath sounds indicates that there is accumulation of secretions in the lower airways. This would be consistent with a nursing diagnosis of ineffective airway clearance because the patient is retaining secretions. 161) Which of the following clinical manifestations would the nurse expect to find during assessment of a patient admitted with pneumococcal pneumonia? A. Hyperresonance on percussion B. Fine crackles in all lobes on auscultation C. Increased vocal fremitus on palpation D. Vesicular breath sounds in all lobes C. Increased vocal fremitus on palpation. A typical physical examination finding for a patient with pneumonia is increased vocal fremitus on palpation. Other signs of pulmonary consolidation include dullness to percussion, bronchial breath sounds, and crackles in the affected area. 162) Which of the following nursing interventions is of the highest priority in helping a patient expectorate thick secretions related to pneumonia? A. Humidify the oxygen as able B. Increase fluid intake to 3L/day if tolerated. C. Administer cough suppressant q4hr. D. Teach patient to splint the affected area. B. Increase fluid intake to 3L/day if tolerated. Although several interventions may help the patient expectorate mucus, the highest priority should be on increasing fluid intake, which will liquefy the secretions so that the patient can expectorate them more easily. Humidifying the oxygen is also helpful, but is not the primary intervention. Teaching the patient to splint the affected area may also be helpful, but does not liquefy the secretions so that they can be removed. 163) During discharge teaching for a 65-year-old patient with emphysema and pneumonia, which of the following vaccines should the nurse recommend the patient receive? A. S. aureus B. H. influenzae C. Pneumococcal D. Bacille Calmette-Guérin (BCG) C. Pneumococcal The pneumococcal vaccine is important for patients with a history of heart or lung disease, recovering from a severe illness, age 65 or over, or living in a long-term care facility. 164) The nurse evaluates that discharge teaching for a patient hospitalized with pneumonia has been most effective when the patient states which of the following measures to prevent a relapse? A. "I will increase my food intake to 2400 calories a day to keep my immune system well." B. "I must use home oxygen therapy for 3 months and then will have a chest x-ray to reevaluate." C. "I will seek immediate medical treatment for any upper respiratory infections." D. "I should continue to do deep-breathing and coughing exercises for at least 6 weeks." D. "I should continue to do deep-breathing and coughing exercises for at least 6 weeks." It is important for the patient to continue with coughing and deep breathing exercises for 6 to 8 weeks until all of the infection has cleared from the lungs. A patient should seek medical treatment for upper respiratory infections that persist for more than 7 days. Increased fluid intake, not caloric intake, is required to liquefy secretions. Home O2 is not a requirement unless the patient's oxygenation saturation is below normal. 165) After admitting a patient to the medical unit with a diagnosis of pneumonia, the nurse will verify that which of the following physician orders have been completed before administering a dose of cefotetan (Cefotan) to the patient? A. Serum laboratory studies ordered for AM B. Pulmonary function evaluation C. Orthostatic blood pressures D. Sputum culture and sensitivity D. Sputum culture and sensitivityThe nurse should ensure that the sputum for culture and sensitivity was sent to the laboratory before administering the cefotetan. It is important that the organisms are correctly identified (by the culture) before their numbers are affected by the antibiotic; the test will also determine whether the proper antibiotic has been ordered (sensitivity testing). Although antibiotic administration should not be unduly delayed while waiting for the patient to expectorate sputum, all of the other options will not be affected by the administration of antibiotics. 166) Which of the following nursing interventions is most appropriate to enhance oxygenation in a patient with unilateral malignant lung disease? A. Positioning patient on right side. B. Maintaining adequate fluid intake C. Performing postural drainage every 4 hours D. Positioning patient with "good lung down" D. Positioning patient with "good lung down" Therapeutic positioning identifies the best position for the patient assuring stable oxygenation status. Research indicates that positioning the patient with the unaffected lung (good lung) dependent best promotes oxygenation in patients with unilateral lung disease. For bilateral lung disease, the right lung down has best ventilation and perfusion. Increasing fluid intake and performing postural drainage will facilitate airway clearance, but positioning is most appropriate to enhance oxygenation. 167) A 71-year-old patient is admitted with acute respiratory distress related to cor pulmonale. Which of the following nursing interventions is most appropriate during admission of this patient? A. Delay any physical assessment of the patient and review with the family the patient's history of respiratory problems. B. Perform a comprehensive health history with the patient to review prior respiratory problems. C. Perform a physical assessment of the respiratory system and ask specific questions related to this episode of respiratory distress. D. Complete a full physical examination to determine the effect of the respiratory distress on other body functions. C. Perform a physical assessment of the respiratory system and ask specific questions related to this episode of respiratory distress.Because the patient is having respiratory difficulty, the nurse should ask specific questions about this episode and perform a physical assessment of this system. Further history taking and physical examination of other body systems can proceed once the patient's acute respiratory distress is being managed. 168) When planning appropriate nursing interventions for a patient with metastatic lung cancer and a 60-pack-year history of cigarette smoking, the nurse recognizes that the smoking has most likely decreased the patient's underlying respiratory defenses because of impairment of which of the following? A. Reflex bronchoconstriction B. Ability to filter particles from the air C. Cough reflex D. Mucociliary clearance D. Mucociliary clearance Smoking decreases the ciliary action in the tracheobronchial tree, resulting in impaired clearance of respiratory secretions, chronic cough, and frequent respiratory infections. 169) While ambulating a patient with metastatic lung cancer, the nurse observes a drop in oxygen saturation from 93% to 86%. Which of the following nursing interventions is most appropriate based upon these findings? A. Continue with ambulation as this is a normal response to activity. B. Move the oximetry probe from the finger to the earlobe for more accurate monitoring during activity. C. Obtain a physician's order for supplemental oxygen to be used during ambulation and other activity. D. Obtain a physician's order for arterial blood gas determinations to verify the oxygen saturation. C. Obtain a physician's order for supplemental oxygen to be used during ambulation and other activity. An oxygen saturation level that drops below 90% with activity indicates that the patient is not tolerating the exercise and needs to have supplemental oxygen applied. 170) The nurse is caring for a 73-year-old patient who underwent a left total knee arthroplasty. On the third postoperative day, the patient complains of shortness of breath, slight chest pain, and that "something is wrong." Temperature is 98.4o F, blood pressure 130/88, respirations 36, and oxygen saturation 91% on room air. Which of the following should the nurse first suspect as the etiology of this episode? A. Septic embolus from the knee joint B. Pulmonary embolus from deep vein thrombosis C. New onset of angina pectoris D. Pleural effusion related to positioning in the operating room B. Pulmonary embolus from deep vein thrombosis The patient presents the classic symptoms of pulmonary embolus: acute onset of symptoms, tachypnea, shortness of breath, and chest pain. 171) In the case of pulmonary embolus from deep vein thrombosis, which of the following actions should the nurse take first? A. Notify the physician. B. Administer a nitroglycerin tablet sublingually. C. Conduct a thorough assessment of the chest pain. D. Sit the patient up in bed as tolerated and apply oxygen. D. Sit the patient up in bed as tolerated and apply oxygen.The patient's clinical picture is consistent with pulmonary embolus, and the first action the nurse takes should be to assist the patient. For this reason, the nurse should sit the patient up as tolerated and apply oxygen before notifying the physician. 172) The nurse is caring for a postoperative patient with sudden onset of respiratory distress. The physician orders a STAT ventilation-perfusion scan. Which of the following explanations should the nurse provide to the patient about the procedure? A. This test involves injection of a radioisotope to outline the blood vessels in the lungs, followed by inhalation of a radioisotope gas. B. This test will use special technology to examine cross sections of the chest with use of a contrast dye. C. This test will use magnetic fields to produce images of the lungs and chest. D. This test involves injecting contrast dye into a blood vessel to outline the blood vessels of the lungs. A. This test involves injection of a radioisotope to outline the blood vessels in the lungs, followed by inhalation of a radioisotope gas.A ventilation-perfusion scan has two parts. In the perfusion portion, a radioisotope is injected into the blood and the pulmonary vasculature is outlined. In the ventilation part, the patient inhales a radioactive gas that outlines the alveoli. 173) During assessment of a 45-year-old patient with asthma, the nurse notes wheezing and dyspnea. The nurse interprets that these symptoms are related to which of the following pathophysiologic changes? A. Laryngospasm B. Overdistention of the alveoli C. Narrowing of the airway D. Pulmonary edema C. Narrowing of the airwayNarrowing of the airway leads to reduced airflow, making it difficult for the patient to breathe and producing the characteristic wheezing. 174) A 45-year-old man with asthma is brought to the emergency department by automobile. He is short of breath and appears frightened. During the initial nursing assessment, which of the following clinical manifestations might be present as an early symptom during an exacerbation of asthma? A. Anxiety B. Cyanosis C. Hypercapnia D. Bradycardia A. Anxiety An early symptom during an asthma attack is anxiety because he is acutely aware of the inability to get sufficient air to breathe. He will be hypoxic early on with decreased PaCO2 and increased pH as he is hyperventilating. 175) The nurse is assigned to care for a patient who has anxiety and an exacerbation of asthma. Which of the following is the primary reason for the nurse to carefully inspect the chest wall of this patient? A. Observe for signs of diaphoresis B. Allow time to calm the patient C. Monitor the patient for bilateral chest expansion D. Evaluate the use of intercostal muscles D. Evaluate the use of intercostal muscles The nurse physically inspects the chest wall to evaluate the use of intercostal (accessory) muscles, which gives an indication of the degree of respiratory distress experienced by the patient. 176) Which of the following positions is most appropriate for the nurse to place a patient experiencing an asthma exacerbation? A. Supine B. Lithotomy C. High-Fowler's D. Reverse Trendelenburg C. High-Fowler'sThe patient experiencing an asthma attack should be placed in high-Fowler's position to allow for optimal chest expansion and enlist the aid of gravity during inspiration. 177) The nurse is caring for a patient with an acute exacerbation of asthma. Following initial treatment, which of the following findings indicates to the nurse that the patient's respiratory status is improving? A. Wheezing becomes louder B. Vesicular breath sounds decrease C. Aerosol bronchodilators stimulate coughing D. The cough remains nonproductive A. Wheezing becomes louder The primary problem during an exacerbation of asthma is narrowing of the airway and subsequent diminished air exchange. As the airways begin to dilate, wheezing gets louder because of better air exchange. 178) The nurse identifies the nursing diagnosis of activity intolerance for a patient with asthma. The nurse assesses for which of the following etiologic factor for this nursing diagnosis in patients with asthma? A. Anxiety and restlessness B. Effects of medications C. Fear of suffocation D. Work of breathing D. Work of breathingWhen the patient does not have sufficient gas exchange to engage in activity, the etiologic factor is often the work of breathing. When patients with asthma do not have effective respirations, they use all available energy to breathe and have little left over for purposeful activity. 179) The nurse is assigned to care for a patient in the emergency department admitted with an exacerbation of asthma. The patient has received a β-adrenergic bronchodilator and supplemental oxygen. If the patient's condition does not improve, the nurse should anticipate which of the following is likely to be the next step in treatment? A. Pulmonary function testing B. Systemic corticosteroids C. Biofeedback therapy D. Intravenous fluids B. Systemic corticosteroids Systemic corticosteroids speed the resolution of asthma exacerbations and are indicated if the initial response to the β-adrenergic bronchodilator is insufficient. 180) A patient with acute exacerbation of COPD needs to receive precise amounts of oxygen. Which of the following types of equipment should the nurse prepare to use? A. Venturi mask B. Partial non-rebreather mask C. Oxygen tent D. Nasal cannula A. Venturi mask The Venturi mask delivers precise concentrations of oxygen and should be selected whenever this is a priority concern. The other methods are less precise in terms of amount of oxygen delivered. 181) While teaching a patient with asthma about the appropriate use of a peak flow meter, the nurse instructs the patient to do which of the following? A. Use the flow meter each morning after taking medications to evaluate their effectiveness. B. Empty the lungs and then inhale quickly through the mouthpiece to measure how fast air can be inhaled. C. Keep a record of the peak flow meter numbers if symptoms of asthma are getting worse. D. Increase the doses of the long-term control medication if the peak flow numbers decrease. C. Keep a record of the peak flow meter numbers if symptoms of asthma are getting worse. It is important to keep track of peak flow readings daily and when the patient's symptoms are getting worse. The patient should have specific directions as to when to call the physician based on personal peak flow numbers. Peak flow is measured by exhaling into the meters and should be assessed before and after medications to evaluate their effectiveness. 182) The physician has prescribed salmeterol (Serevent) for a patient with asthma. In reviewing the use of dry powder inhalers (DPIs) with the patient, the nurse should provide which of the following instructions? A. "Close lips tightly around the mouthpiece and breathe in deeply and quickly." B. "To administer a DPI, you must use a spacer that holds the medicine so that you can inhale it." C. "Hold the inhaler several inches in front of your mouth and breathe in slowly, holding the medicine as long as possible." D. "You will know you have correctly used the DPI when you taste or sense the medicine going into your lungs." A. "Close lips tightly around the mouthpiece and breathe in deeply and quickly." Dry powder inhalers do not require spacer devices. The patient should be instructed to breathe in deeply and quickly to ensure medicine moves down deeply into lungs. The patient may not taste or sense the medicine going into the lungs. 183) The nurse determines that a patient is experiencing common adverse effects from the inhaled corticosteroid beclomethasone (Beclovent) after noting which of the following? A. Adrenocortical dysfunction and hyperglycemia B. Elevation of blood glucose and calcium levels C. Oropharyngeal candidiasis and hoarseness D. Hypertension and pulmonary edema C. Oropharyngeal candidiasis and hoarseness Oropharyngeal candidiasis and hoarseness are common adverse effects from the use of inhaled corticosteroids because the medication can lead to overgrowth of organisms and local irritation if the patient does not rinse the mouth following each dose. 184) The nurse determines that the patient understood medication instructions about the use of a spacer device when taking inhaled medications after hearing the patient state which of the following as the primary benefit? A. "Now I will not need to breathe in as deeply when taking the inhaler medications." B. "This device will make it so much easier and faster to take my inhaled medications." C. "I will pay less for medication because it will last longer." D. "More of the medication will get down into my lungs to help my breathing." D. "More of the medication will get down into my lungs to help my breathing." A spacer assists more medication to reach the lungs, with less being deposited in the mouth and the back of the throat. 185) Which of the following test results identify that a patient with an asthma attack is responding to treatment? A. A decreased exhaled nitric oxide B. An increase in CO2 levels C. A decrease in white blood cell count D. An increase in serum bicarbonate levels A. A decreased exhaled nitric oxide. Nitric oxide levels are increased in the breath of people with asthma. A decrease in the exhaled nitric oxide concentration suggests that the treatment may be decreasing the lung inflammation associated with asthma. 186) The nurse determines that the patient is not experiencing adverse effects of albuterol (Proventil) after noting which of the following patient vital signs? A. Oxygen saturation 96% B. Respiratory rate of 18 C. Temperature of 98.4° F D. Pulse rate of 76 D. Pulse rate of 76 Albuterol is a β2-agonist that can sometimes cause adverse cardiovascular effects. These would include tachycardia and angina. A pulse rate of 76 indicates that the patient did not experience tachycardia as an adverse effect. 187) The patient has an order for each of the following inhalers. Which of the following should the nurse offer to the patient at the onset of an asthma attack? A. Albuterol (Proventil) B. Beclomethasone (Beclovent) C. Ipratropium bromide (Atrovent) D. Salmeterol (Serevent) A. Albuterol (Proventil) Albuterol is a short-acting bronchodilator that should initially be given when the patient experiences an asthma attack. 188) The nurse who has administered a first dose of oral prednisone (Deltasone) to the patient with asthma writes on the care plan to begin monitoring which of the following patient parameters? A. Intake and output B. Bowel sounds C. Apical pulse D. Deep tendon reflexes A. Intake and output Corticosteroids such as prednisone can lead to fluid retention. For this reason, it is important to monitor the patient's intake and output. 189) The nurse is assisting a patient to learn self-administration of beclomethasone two puffs inhalation q6hr. The nurse explains that the best way to prevent oral infection while taking this medication is to do which of the following as part of the self-administration techniques? A. Chew a hard candy before the first puff of medication. B. Ask for a breath mint following the second puff of medication. C. Rinse the mouth with water before each puff of medication. D. Rinse the mouth with water following the second puff of medication. D. Rinse the mouth with water following the second puff of medication. The patient should rinse the mouth with water following the second puff of medication to reduce the risk of fungal overgrowth and oral infection. 190) The nurse is scheduled to give a dose of salmeterol by metered dose inhaler (MDI). The nurse would administer the right drug by selecting the inhaler with which of the following trade names? A. Vanceril B. Serevent C. AeroBid D. Atrovent B. Serevent The trade or brand name for salmeterol, an adrenergic bronchodilator, is Serevent. 191) The nurse is evaluating whether a patient understands how to safely determine whether a metered dose inhaler is empty. The nurse interprets that the patient understands this important information to prevent medication underdosing when the patient describes which method to check the inhaler? A. Place it in water to see if it floats. B. Shake the canister while holding it next to the ear. C. Check the indicator line on the side of the canister. D. Keep track of the number of inhalations used. D. Keep track of the number of inhalations used. It is no longer appropriate to see if a canister floats in water or not as research has demonstrated this is not accurate. The best method to determine when to replace an inhaler is by knowing the maximum puffs available per MDI and then replacing when those inhalations have been used. 192) The nurse is scheduled to give a dose of ipratropium bromide by metered dose inhaler. The nurse would administer the right drug by selecting the inhaler with which of the following trade names? A. Vanceril B. Pulmicort C. AeroBid D. Atrovent D. Atrovent The trade or brand name for ipratropium bromide, an anticholinergic medication, is Atrovent. 193) The patient has an order for albuterol 5 mg via nebulizer. Available is a solution containing 2 mg/ml. How many milliliters should the nurse use to prepare the patient's dose? A. 0.2 B. 2.5 C. 3.75 D. 5.0 B. 2.5 194) When planning patient teaching about emphysema, the nurse understands that the symptoms of emphysema are caused by which of the following? A. Hypertrophy and hyperplasia of goblet cells in the bronchi B. Collapse and hypoventilation of the terminal respiratory unit C. An overproduction of the antiprotease alpha1-antitrypsin D. Hyperinflation of alveoli and destruction of alveolar walls D. Hyperinflation of alveoli and destruction of alveolar walls In emphysema, there are structural changes that include hyperinflation of alveoli, destruction of alveolar walls, destruction of alveolar capillary walls, narrowing of small airways, and loss of lung elasticity. 195) The patient has an order for albuterol 5 mg via nebulizer. Available is a solution containing 1 mg/ml. How many milliliters should the nurse use to prepare the patient's dose? A. 0.2 B. 2.5 C. 3.75 D. 5.0 D. 5.0 196) The nurse evaluates that nursing interventions to promote airway clearance in a patient admitted with COPD are successful based on which of the following findings? A. Absence of dyspnea B. Improved mental status C. Effective and productive coughing D. PaO2 within normal range for the patient C. Effective and productive coughing The issue of the question is airway clearance, which is most directly evaluated as successful if the patient can engage in effective and productive coughing. 197) When caring for a patient with COPD, the nurse identifies a nursing diagnosis of imbalanced nutrition less than body requirements after noting a weight loss of 30 lb. Which of the following would be an appropriate intervention to add to the plan of care for this patient? A. Teach the patient to use frozen meals at home that can be microwaved. B. Provide a high-calorie, high-carbohydrate, nonirritating, frequent feeding diet. C. Order fruits and fruit juices to be offered between meals. D. Order a high-calorie, high-protein diet with six small meals a day. D. Order a high-calorie, high-protein diet with six small meals a day.Because the patient with COPD needs to use greater energy to breathe, there is often decreased oral intake because of dyspnea. A full stomach also impairs the ability of the diaphragm to descend during inspiration, interfering with the work of breathing. Finally, the metabolism of a high carbohydrate diet yields large amounts of CO2, which may lead to acidosis in patients with pulmonary disease. For these reasons, the patient with emphysema should take in a high-calorie, high-protein diet, eating six small meals per day. 198) The nurse reviews pursed lip breathing with a patient newly diagnosed with emphysema. The nurse reinforces that this technique will assist respiration by which of the following mechanisms? A. Preventing bronchial collapse and air trapping in the lungs during exhalation B. Increasing the respiratory rate and giving the patient control of respiratory patterns C. Loosening secretions so that they may be coughed up more easily D. Promoting maximal inhalation for better oxygenation of the lungs A. Preventing bronchial collapse and air trapping in the lungs during exhalation The focus of pursed lip breathing is to slow down the exhalation phase of respiration, which decreases bronchial collapse and subsequent air trapping in the lungs during exhalation. 199) Nursing assessment findings of jugular vein distention and pedal edema would be indicative of which of the following complications of emphysema? A. Acute respiratory failure B. Pulmonary edema caused by left-sided heart failure C. Fluid volume excess secondary to cor pulmonale D. Secondary respiratory infection C. Fluid volume excess secondary to cor pulmonale Cor pulmonale is a right-sided heart failure caused by resistance to right ventricular outflow due to lung disease. With failure of the right ventricle, the blood emptying into the right atrium and ventricle would be slowed, leading to jugular venous distention and pedal edema. 200) A patient has been receiving oxygen per nasal cannula while hospitalized for COPD. The patient asks the nurse whether oxygen use will be needed at home. Which of the following would be the most appropriate response by the nurse? A. "Long-term home oxygen therapy should be used to prevent respiratory failure." B. "Oxygen will be needed when your oxygen saturation drops to 88% and you have symptoms of hypoxia. C. "Long-term home oxygen therapy should be used to prevent heart problems related to emphysema." D. "Oxygen will not be needed until or unless you are in the terminal stages of this disease." B. "Oxygen will be needed when your oxygen saturation drops to 88% and you have symptoms of hypoxia.Long-term oxygen therapy in the home should be considered when the oxygen saturation is 88% or less and the patient has signs of tissue hypoxia, such as cor pulmonale, erythrocytosis, or impaired mental status. 201) Before discharge, the nurse discusses activity levels with a 61-year-old patient with COPD and pneumonia. Which of the following exercise goals is most appropriate once the patient is fully recovered from this episode of illness? A. Slightly increase activity over the current level. B. Walk for 20 minutes a day, keeping the pulse rate less than 130 beats per minute. C. Limit exercise to activities of daily living to conserve energy. D. Swim for 10 min/day, gradually increasing to 30 min/day. B. Walk for 20 minutes a day, keeping the pulse rate less than 130 beats per minute. The patient will benefit from mild aerobic exercise that does not stress the cardiorespiratory system. The patient should be encouraged to walk for 20 min/day, keeping the pulse rate less than 75% to 80% of maximum heart rate (220 minus patient's age). 202) The nurse evaluates that a patient is experiencing the expected beneficial effects of ipratropium (Atrovent) after noting which of the following assessment findings? A. Increased peak flow readings B. Increased level of consciousness C. Decreased sputum production D. Increased respiratory rate A. Increased peak flow readings. Ipratropium is a bronchodilator that should lead to increased PEFRs. 203) The nurse is teaching a patient how to self-administer ipratropium (Atrovent) via a metered dose inhaler. Which of the following instructions given by the nurse is most appropriate to help the patient learn proper inhalation technique? A. "Avoid shaking the inhaler before use." B. "Breathe out slowly before positioning the inhaler." C. "After taking a puff, hold the breath for 30 seconds before exhaling." D. "Using a spacer should be avoided for this type of medication." B. "Breathe out slowly before positioning the inhaler." It is important to breathe out slowly before positioning the inhaler. This allows the patient to take a deeper breath while inhaling the medication thus enhancing the effectiveness of the dose. 204) Which of the following statements made by a patient with COPD indicates a need for further education regarding the use of an ipratropium inhaler? A. "I should rinse my mouth following the two puffs to get rid of the bad taste." B. "I should wait at least 1 to 2 minutes between each puff of the inhaler." C. "If my breathing gets worse, I should keep taking extra puffs of the inhaler until I can breathe more easily." D. "Because this medication is not fast-acting, I cannot use it in an emergency if my breathing gets worse. C. "If my breathing gets worse, I should keep taking extra puffs of the inhaler until I can breathe more easily." The patient should not take extra puffs of the inhaler at will to make breathing easier. Excessive treatment could trigger paradoxical bronchospasm, which would worsen the patient's respiratory status. 205) When assessing a patient's sleep-rest pattern related to respiratory health, the nurse would ask if the patient: (Select all that apply.) A. Has trouble falling asleep B. Awakens abruptly during the night C. Sleeps more than 8 hours per night D. Has to sleep with the head elevated A,B,D The patient with sleep apnea may have insomnia and/or abrupt awakenings. Patients with cardiovascular disease (e.g., heart failure that may affect respiratory health) may need to sleep with the head elevated on several pillows (orthopnea). Sleeping more than 8 hours per night is not indicative of impaired respiratory health. 206) A patient is being discharged from the emergency department after being treated for epistaxis. In teaching the family first aid measures in the event the epistaxis would recur, which of the following measures would the nurse suggest? (Select all that apply.) A. Tilt patients head backwards B. Apply ice compresses to the nose C. Pinch the entire soft lower portion of the nose D. Partially insert a small gauze pad into the bleeding nostril B,C,D First aid measures to control epistaxis includes placing the patient in a sitting position, leaning forward. Tilting the head back does not stop the bleeding, but rather allows the blood to enter the nasopharynx, which could result in aspiration or nausea/vomiting from swallowing blood. All of the other options are appropriate first aid treatment of epistaxis. 207) To promote airway clearance in a patient with pneumonia, the nurse instructs the patient to do which of the following? (Select all that apply.) A. Splint the chest when coughing B. Maintain a semi-Fowler's position C. Maintain adequate fluid intake D. Instruct patient to cough at end of exhalation A,C,D The nurse should instruct the patient to splint the chest while coughing. This will reduce discomfort and allow for a more effective cough. Maintaining adequate fluid intake liquefies secretions, allowing easier expectoration. Coughing at the end of exhalation promotes a more effective cough. The patient should be positioned in an upright sitting position (high-Fowler's) with head slightly flexed. 208) During admission of a patient diagnosed with non-small cell carcinoma of the lung, the nurse questions the patient related to a history of which of the following risk factors for this type of cancer? (Select all that apply.) A. Asbestos exposure B. Cigarette smoking C. Exposure to uranium D. Chronic interstitial fibrosis A,B,C Non-small carcinoma is associated with cigarette smoking and exposure to environmental carcinogens, including asbestos and uranium. Chronic interstitial fibrosis is associated with the development of adenocarcinoma of the lung. 209) When admitting a 45-year-old female with a diagnosis of pulmonary embolism, the nurse will assess the patient for which of the following risk factors? (Select all that apply.) A. Obesity B. Pneumonia C. Hypertension D. Cigarette smoking A,C,D Research has demonstrated an increased risk of pulmonary embolism in women associated with obesity, heavy cigarette smoking, and hypertension. Other risk factors include immobilization, surgery within the last 3 months, stroke, history of DVT, and malignancy. 210) When admitting a patient with the diagnosis of asthma exacerbation, the nurse will assess for which of the following potential triggers? (Select all that apply.) A. Exercise B. Allergies C. Emotional stress D. Decreased humidity A,B,C Although the exact mechanism of asthma is unknown, there are several triggers that may precipitate an attack. These include allergens, exercise, air pollutants, respiratory infections, drug and food additives, psychologic factors, and GERD. 211) The arterial blood gas (ABG) readings that indicate compensated respiratory acidosis are a PaCO2 of A. 30 mm Hg and bicarbonate level of 24 mEq/L. B. 30 mm Hg and bicarbonate level of 30 mEq/L. C. 50 mm Hg and bicarbonate level of 20 mEq/L. D. 50 mm Hg and bicarbonate level of 30 mEq/L. D. 50 mm Hg and bicarbonate level of 30 mEq/L. If compensation is present, carbon dioxide and bicarbonate are abnormal (or nearly so) in opposite directions (e.g., one is acidotic and the other alkalotic). 212) A patient admitted to the emergency department with tension pneumothorax and mediastinal shift following an automobile crash is most likely to exhibit A. bradycardia. B. severe hypotension. C. mediastinal flutter. D. a sucking chest wound. B. severe hypotension. Mediastinal shift may cause compression of the lung in the direction of the shift and compression, traction, torsion, or kinking of the great vessels. Blood return to the heart is dangerously impaired and causes a subsequent decrease in cardiac output and blood pressure. Tachycardia is a clinical manifestation of tension pneumothorax. An uncovered opened pneumothorax is associated with a sucking chest wound and mediastinal flutter. 213) In preparing the preoperative teaching plan for a patient who is to undergo a total laryngectomy, a nurse should give highest priority to the A. tracheostomy being in place for 2 to 3 days. B. patient's not being able to speak normally again. C. insertion of a gastrostomy feeding tube during surgery. D. patient's not being able to perform deep-breathing exercises. B. patient's not being able to speak normally again. Patients who have a total laryngectomy have a permanent tracheostomy and will need to learn how to speak using alternative methods, such as an artificial larynx. The tracheostomy will be permanent to allow normal breathing patterns and air exchange. After surgery, the patient's nutrition is supplemented with enteral feedings, and when the patient can swallow secretions, oral feedings can begin. Deep-breathing exercises should be performed with the patient at least every 2 hours to prevent further pulmonary complications. 214) After a posterior nasal pack is inserted by a physician, the patient is very anxious and states, "I don't feel like I'm breathing right." The immediate intervention the nurse should initiate is to A. monitor ABGs. B. reassure the patient that this is normal discomfort. C. cut the pack strings and pull the packing out with a hemostat. D. direct a flashlight into the patient's mouth and inspect the oral cavity. D. direct a flashlight into the patient's mouth and inspect the oral cavity. The nurse should inspect the oral cavity for the presence of blood, soft palate necrosis, and proper placement of the posterior plug. If the posterior plug is visible, the physician should be notified for readjustment of the packing. Reassurance, cutting the strings, and ABGs are not top priority interventions. The nurse needs further data before intervening. 215) A nurse is performing assessment for a patient diagnosed with chronic obstructive pulmonary disease (COPD). Which of the following findings should the nurse expect to observe? A. Nonproductive cough B. Prolonged inspiration C. Vesicular breath sounds D. Increased anterior-posterior chest diameter D. Increased anterior-posterior chest diameter An increased anterior-posterior diameter is a compensatory mechanism experienced by patients with COPD and is caused by air-trapping. Patients with COPD have a productive cough, often expectorating copious amounts of sputum. Because of air-trapping, patients with COPD experience a prolonged expiration because the rate of gas on exhalation takes longer to escape. Chest auscultation for patients with COPD often reveals wheezing, crackles, and other adventitious breath sounds. 216) A nurse is working on a respiratory care unit where many of the patients are affected by asthma. Which of the following actions by the nurse would most likely increase respiratory difficulty for the patients? A. Wearing perfume to work B. Encouraging patients to ambulate daily C. Allowing the patients to eat green leafy vegetables D. Withholding antibiotic therapy until cultures are obtained A. Wearing perfume to work People with asthma should avoid extrinsic allergens and irritants (e.g., dust, pollen, smoke, certain foods, colognes and perfumes, certain types of medications) because their airways become inflamed, producing shortness of breath, chest tightness, and wheezing. Many green leafy vegetables are rich in vitamins, minerals, and proteins, which incorporate healthy lifestyle patterns into the patients' daily living routines. Routine exercise is a part of a prudent lifestyle, and for patients with asthma the physical and psychosocial effects of ambulation can incorporate feelings of well-being, strength, and enhancement of physical endurance. Antibiotic therapy is always initiated after cultures are obtained so that the sensitivity to the organism can be readily identified. 217) The most appropriate position to assist a patient with chronic obstructive pulmonary disease (COPD) who is having difficulty breathing would be a A. high Fowler's position without a pillow behind the head. B. semi-Fowler's position with a single pillow behind the head. C. right side-lying position with the head of the bed at 45 degrees' elevation. D. sitting upright and forward position with arms supported on an over-the-bed table. D. sitting upright and forward position with arms supported on an over-the-bed table.Sitting upright and leaning forward with arms supported on an over-the-bed table would be of most help to this patient, because it allows for expansion of the thoracic cage in all four directions (front, back, and two sides). 218) A person complains of fatigue and malaise and has a slight temperature elevation for 2 days before symptoms of influenza (fever, chest congestion, and productive cough) become noticeable. During the time immediately before the illness is diagnosed, the patient A. could avoid contracting the disease if treatment is begun with antibiotics. B. is unable to spread the disease because it is still in the incubation period. C. is in the prodromal stage and is highly contagious and able to spread the disease. D. has a nosocomial infection, which affects approximately two million individuals a year. C. is in the prodromal stage and is highly contagious and able to spread the disease. The prodromal stage is a short period of time (hours to several days) immediately preceding the onset of an illness during which the patient is very contagious. Antibiotics are not effective against viral illnesses. The incubation period is the time from entry of the organism to the onset of symptoms and, in some viral illnesses, may be contagious. Nosocomial infections are those acquired in a hospital, and this scenario does not suggest the source of the infection. 219) In older adults, infection after exposure to respiratory illness is most likely to A. result in similar rates of infection as in the younger adult. B. be easily prevented with the use of antibiotics after being exposed. C. result in serious lower respiratory infection related to weakened respiratory muscles and fewer cilia. D. be less serious because the older adult has less contact with younger children who are most likely to carry serious infections. C. result in serious lower respiratory infection related to weakened respiratory muscles and fewer cilia. Changes in the older adult respiratory system make older adults more susceptible to infections that can be very serious and life threatening. Use of antibiotics to "prevent" lung infections is not recommended and is ineffective for viral infections. 220) If a nurse is caring for an 80-year-old patient with a temperature of 100.4° F, crackles at the right lung base, pain with deep inspiration, and dyspnea, which of the following orders is the nurse's priority? A. Sputum specimen for culture and sensitivity B. Codeine 15 mg orally every 6 hours as needed C. Incentive spirometer every 2 hours while awake D. Amoxicillin (Amoxil) 500 mg orally 4 times a day A. Sputum specimen for culture and sensitivity The patient presents with signs of a respiratory infection. To initiate the most effective therapy, the health care prescriber must know the pathogen causing the infection. Therefore, the sputum specimen is the nurse's priority. If the antibiotic is administered before the specimen is obtained, the results of the culture might not be as accurate and could impair the effectiveness of therapy. After the specimen is obtained, the nurse can administer codeine for coughing and begin the incentive spirometry to mobilize secretions and improve the patient's ability to expectorate the secretions. 221) When assessing a patient's respiratory status, which of the following nonrespiratory data are most important for the nurse to obtain? A. Height and weight B. Neck circumference C. Occupation and hobbies D. Usual daily fluid intake C. Occupation and hobbiesMany respiratory problems occur as a result of chronic exposure to inhalation irritants. Common occupational sources of inhalation irritants include mines, granaries, farms, lawn care companies, paint, plastics and rubber manufacture, and building remodeling. Hobbies associated with inhalation irritants include woodworking, metal finishing, furniture refinishing, painting, and ceramics. Daily fluids, height, and weight are more related to respiratory problems secondary to cardiac issues. 222) If a nurse is assessing a patient whose recent blood gas determination indicated a pH of 7.32 and respirations are measured at 32 breaths/min, which of the following is the most appropriate nursing assessment? A. The rapid breathing is causing the low pH. B. The nurse should sedate the patient to slow down respirations. C. The rapid breathing is an attempt to compensate for the low pH. D. The nurse should give the patient a paper bag to breathe into to correct the low pH. C. The rapid breathing is an attempt to compensate for the low pH. The respiratory system influences pH (acidity) through control of carbon dioxide exhalation. Thus, rapid breathing increases the pH. Breathing into a paper bag aids a patient who is hyperventilating; in respiratory alkalosis, it aids in lowering the pH. The use of sedation can cause respiratory depression and hypoventilation, resulting in an even lower pH. 223) If a patient with an uncuffed tracheostomy tube coughs violently during suctioning and dislodges the tracheostomy tube, a nurse should first A. call the physician. B. attempt to reinsert the tracheostomy tube. C. position the patient in a lateral position with the neck extended. D. cover the stoma with a sterile dressing and ventilate the patient with a manual bag-mask until the physician arrives. B. attempt to reinsert the tracheostomy tube.Retention sutures may be grasped (if present) and the tracheostomy opening spread, or a hemostat may be used to spread the opening. The obturator is inserted into the replacement tube (one size smaller than the original tube), lubricated with saline solution, and inserted into the stoma at a 45-degree angle to the neck. If the attempt is successful, the obturator tube should immediately be removed. 224) Upon entering the room of a patient who has just returned from surgery for total laryngectomy and radical neck dissection, a nurse should recognize a need for intervention when finding A. a gastrostomy tube that is clamped. B. the patient coughing blood-tinged secretions from the tracheostomy. C. the patient positioned in a lateral position with the head of the bed flat. D. 200 ml of serosanguineous drainage in the patient's portable drainage device. C. the patient positioned in a lateral position with the head of the bed flat. After total laryngectomy and radical neck dissection, a patient should be placed in a semi-Fowler's position to decrease edema and limit tension on the suture line. 225) When administering oxygen to a patient with COPD with the potential for carbon dioxide narcosis, the nurse should A. never administer oxygen at a rate of more than 2 L/min. B. monitor the patient's use of oxygen to detect oxygen dependency. C. monitor the patient for symptoms of oxygen toxicity, such as paresthesias. D. use ABGs as a guide to determine what FIO2 level meets the patient's needs. D. use ABGs as a guide to determine what FIO2 level meets the patient's needs. It is critical to start oxygen at low flow rates and then use ABGs as a guide to determine what FIO2 level is sufficient and can be tolerated. 226) To ensure the correct amount of oxygen delivery for a patient receiving 35% oxygen via a Venturi mask, it is most important that the nurse A. keep the air-entrainment ports clean and unobstructed. B. apply an adaptor to increase humidification of the oxygen. C. drain moisture condensation from the oxygen tubing every hour. D. keep the flow rate high enough to keep the bag from collapsing during inspiration. A. keep the air-entrainment ports clean and unobstructed. Oxygen is delivered to a small jet in the center of a wide-based cone. Air is entrained (pulled through) openings in the cone as oxygen flows through the small jet. The degree of restriction or narrowness of the jet determines the amount of entrainment and the dilution of pure oxygen with room air and thus the concentration of oxygen. Although applying an adaptor can increase the humidification with the Venturi mask, it is not the best answer, because an open port is essential to proper functioning. Draining moisture condensation from the oxygen tubing is performed as often as needed, not on an hourly schedule. A plastic face mask with a reservoir bag needs to have sufficient flow rate to keep the bag inflated. 227) While caring for a patient with respiratory disease, a nurse observes that the oxygen saturation drops from 94% to 85% when the patient ambulates. The nurse should determine that A. supplemental oxygen should be used when the patient exercises. B. ABG determinations should be done to verify the oxygen saturation reading. C. this finding is a normal response to activity and that the patient should continue to be monitored. D. the oximetry probe should be moved from the finger to the earlobe for an accurate oxygen saturation measurement during activity. A. supplemental oxygen should be used when the patient exercises.An oxygen saturation lower than 90% indicates inadequate oxygenation. If the drop is related to activity of some type, supplemental oxygen is indicated. 228) A nurse establishes the presence of a tension pneumothorax when assessment findings reveal a(n) A. absence of lung sounds on the affected side. B. inability to auscultate tracheal breath sounds. C. deviation of the trachea toward the side opposite the pneumothorax. D. shift of the point of maximal impulse (PMI) to the left, with bounding pulses. C. deviation of the trachea toward the side opposite the pneumothorax. Tension pneumothorax is caused by rapid accumulation of air in the pleural space, causing severely high intrapleural pressure. This results in collapse of the lung, and the mediastinum shifts toward the unaffected side, which is subsequently compressed. 229) Which of the following statements made by a nurse would indicate proper teaching principles regarding feeding and tracheostomies? A. "Follow each spoon of food consumed with a drink of fluid." B. "Thin your foods to a liquid consistency whenever possible." C. "Tilt your chin forward toward the chest when swallowing your food." D. "Make sure your cuff is overinflated before eating if you have swallowing problems." C. "Tilt your chin forward toward the chest when swallowing your food." A nurse should instruct a patient to tilt the chin toward the chest, which will close the glottis and allow food to enter the normal passageway. Ideally, foods should be of a thick consistency to enable effective swallowing and reduce the risk of aspiration. Overinflation of the cuff causes swallowing difficulties. Fluids should be consumed in small amounts after swallowing to prevent the risk of aspiration. 230) If a patient states, "It's hard for me to breathe and I feel short-winded all the time," what is the most appropriate terminology to be applied in documenting this assessment by a nurse? A. Apnea B. Dyspnea C. Tachypnea D. Respiratory fatigue B. Dyspnea Dyspnea is a subjective description reflective of the patient's statement indicating difficulty in breathing. Apnea refers to absence of breath or breathing. Tachypnea refers to an increased rate of breathing, usually greater than 20 breaths per minute. Respiratory fatigue is subjective and usually refers to the patient exhibiting signs and symptoms associated with a comprehensive respiratory assessment including laborious breathing, use of accessory muscles, and slowing of respirations. 231) To prevent atelectasis in an 82-year-old patient with a hip fracture, a nurse should A. supply oxygen. B. suction the upper airway. C. ambulate the patient frequently. D. assist the patient with aggressive coughing and deep breathing. D. assist the patient with aggressive coughing and deep breathing. Decreased mobility after surgery in older adults creates the possibility of fluid buildup and retention in lung tissue. One of the primary goals of nursing intervention is to prevent atelectasis in a high-risk patient. Aggressive coughing and deep breathing can prevent atelectasis in the postoperative patient. 232) Respiratory acidosis is at highest risk in a patient with A. hypokalemia. B. pulmonary fibrosis. C. salicylate overdose. D. COPD. D. COPD. Chronic respiratory acidosis is most commonly caused by COPD. Pulmonary fibrosis, hypokalemia, and salicylate overdose do not predispose a patient to respiratory acidosis. Hypokalemia can lead to cardiac dysrhythmias. Salicylate overdose results in central nervous system changes, and pulmonary fibrosis can result in respiratory arrest. 233) Select all that apply. Atelectasis can be caused by A. long-term smoking. B. inadequate surfactant. C. localized airway obstruction. D. an increase in lung expansion. E. an increase in elastic recoil. BCE The collapse of lung tissue has several causes, including reduced lung expansion, localized airway obstruction, inadequate surfactant, and an increase in elastic recoil. Smoking, although harmful, does not in itself cause atelectasis. 234) A patient is having inspiratory stridor (crowing respiration) and the nurse suspects he is experiencing a laryngospasm. Which of the following would be most appropriate to implement for a patient experiencing a laryngospasm? A. Administer 100% oxygen. B. Position the patient in high Fowler's position. C. Insert a 16-gauge (large-bore) IV needle. D. Activate the emergency response team (code blue team) to the patient's room. A. Administer 100% oxygen.A nurse should immediately administer 100% oxygen to the patient until the airway is fully reestablished, the larynx relaxes, and the spasms stop. Activating the emergency response team is not an immediate nursing action at this time because the nurse can administer the oxygen without the assistance of others. Positioning the patient in high Fowler's will not address the patient's need for immediate reoxygenation because of the patient's compromised respiratory state. Insertion of an IV device is not the first priority response but should be implemented after the nurse has assessed that the airway is stable. 235) A nurse is preparing to establish oxygen therapy for a patient with COPD, and the physician's prescription reads "oxygen per nasal cannula at 5 L per minute." Which of the following actions should the nurse take? A. Administer the oxygen as prescribed. B. Call the physician and question the correct flow rate of the oxygen. C. Establish the oxygen as prescribed and obtain an ABG. D. Change the delivery device from a nasal cannula to a simple oxygen mask. B. Call the physician and question the correct flow rate of the oxygen. The nurse should call the physician immediately and question the flow rate for delivery of the oxygen before implementation. Oxygen is used cautiously in patients with COPD because of longstanding hypoxemia serving as the respiratory drive mechanism. If high levels of oxygen are administered, the respiratory drive can be obliterated. Changing the device to a simple oxygen mask may alter the oxygen concentration being delivered to the patient and will further enhance the obliteration of the patient's respiratory drive. Obtaining an ABG sample is not a priority at this time, and the action does not address the validity of the prescribed oxygen dosing for the patient. 236) A 75-year-old obese patient who is snoring loudly and having periods of apnea several times each night is most likely experiencing A. narcolepsy. B. sleep apnea. C. sleep deprivation. D. paroxysmal nocturnal dyspnea. B. sleep apnea. Sleep apnea is most common in obese patients. Typical symptoms include snoring and periods of apnea. Narcolepsy is when a patient falls asleep unexpectedly. Sleep deprivation could result from sleep apnea. Paroxysmal nocturnal dyspnea occurs when a patient has shortness of breath during the night. 237) Which of the following conditions is manifested by unexplained shortness of breath and a high mortality rate? A. Bleeding ulcer B. Transient ischemia C. Pulmonary embolism D. MI C. Pulmonary embolism A high mortality rate is associated with a pulmonary embolism. A pulmonary embolism is an obstruction of the pulmonary artery caused by an embolus. It presents with hypoxia, anxiety, restlessness, and shortness of breath. Bleeding ulcers, MI, and transient ischemia are not associated with such a high mortality rate. 238) A patient with COPD is receiving oxygen at 2 L/min. While in the supine position for a bath, the patient complains of shortness of breath. What is the most appropriate first nursing action? A. Increase the flow of oxygen. B. Perform tracheal suctioning. C. Report this to the physician. D. Assist the patient to Fowler's position. D. Assist the patient to Fowler's position. Breathing is easier in Fowler's position because it permits greater expansion of the chest cavity. If repositioning does not improve the situation, then oxygenation and physician reporting might be appropriate. The patient would not benefit from tracheal suctioning. 239) To find the infection site associated with acute lymphangitis, the nurse should look to the inflammation. A. distal B. anterior C. proximal D. contralateral A. distal The nurse should assess distal to swelling to locate the initial site of infection. Examining proximal, contralateral, or anterior to the inflammation does not describe swelling associated with infection. 240) Which of the following instructions are most appropriate in the home management of a patient who has undergone surgery for oral cancer? A. "You should drink plenty of fluids and eat foods you enjoy." B. "It is normal to have some leakage of saliva from the suture line." C. "Lying in a prone position helps decrease swelling at the suture line." D. "You should avoid foods high in protein while your suture line is healing." A. "You should drink plenty of fluids and eat foods you enjoy." For patients who have undergone treatment for head and neck cancers, maintaining adequate nutrition is a challenge. The nurse encourages the patient to increase fluids to prevent dehydration and liquefy secretions. These patients are more likely to eat foods that they enjoy and can tolerate. 241) Which of the following conditions or factors in a 64-year-old patient diagnosed with head and neck cancer most likely contributed to this health problem? A. Patient's hobby is oil painting. B. Patient's father also had head and neck cancer. C. Patient uses chewing tobacco and drinks beer daily. D. Patient quit school at age 16 and has worked in a butcher shop for more than 40 years. C. Patient uses chewing tobacco and drinks beer daily. Many environmental risk factors contribute to the development of head and neck cancer, although the actual cause is unknown. There does not appear to be a genetic predisposition to this type of cancer. The two most important risk factors are tobacco and alcohol use, especially in combination. Other risk factors include chewing tobacco, pipe smoking, marijuana use, voice abuse, chronic laryngitis, exposure to industrial chemicals or hardwood dust, and poor oral hygiene. 242) A patient's ABGs include a PaO2 of 88 mm Hg and a PaCO2 of 38 mm Hg and mixed venous blood gases include a PvO2 of 40 mm Hg and PvCO2 of 46 mm Hg. These findings indicate that the patient has A. impaired cardiac output. B. unstable hemodynamics. C. inadequate delivery of oxygen to the tissues. D. normal capillary oxygen-carbon dioxide exchange. D. normal capillary oxygen-carbon dioxide exchange. Normal venous blood gas values reflect the normal uptake of oxygen from arterial blood and the release of carbon dioxide from cells into the blood, resulting in a much lower PaO2 and an increased PaCO2. The pH is also decreased in mixed venous blood gases because of the higher PvCO2. Normal mixed venous blood gases also have much lower PvO2 and SvO2 than arterial blood bases. Mixed venous blood gases are used when patients are hemodynamically unstable to evaluate the amount of oxygen delivered to the tissue and the amount of oxygen consumed by the tissues. 243) An excess of carbon dioxide in the blood causes an increased respiratory rate and volume because CO2 A. displaces oxygen on hemoglobin, leading to a decreased PaO2. B. causes an increase in the amount of hydrogen ions available in the body. C. combines with water to form carbonic acid, lowering the pH of cerebrospinal fluid. D. directly stimulates chemoreceptors in the medulla to increase respiratory rate and volume. C. combines with water to form carbonic acid, lowering the pH of cerebrospinal fluid. A combination of excess CO2 and H2O results in carbonic acid, which lowers the pH of the cerebrospinal fluid and stimulates an increase in the respiratory rate. Peripheral chemoreceptors in the carotid and aortic bodies also respond to increases in PaCO2 to stimulate the respiratory center. Excess CO2 does not increase the amount of hydrogen ions available in the body but does combine with the hydrogen of water to form an acid. 244) A patient with an acute pharyngitis is seen at the clinic with fever and severe throat pain that affects swallowing. On inspection the throat is reddened and edematous with patchy yellow exudates. The nurse anticipates that collaborative management will include A. treatment with antibiotics. B. treatment with antifungal agents. C. a throat culture or rapid strep antigen test. D. treatment with medication only if the pharyngitis does not resolve in 3 to 4 days. C. a throat culture or rapid strep antigen test. Although inadequately treated β-hemolytic streptococcal infections may lead to rheumatic heart disease or glomerulonephritis, antibiotic treatment is not recommended until strep infections are definitely diagnosed with culture or antigen tests. The manifestations of viral and bacterial infections are similar, and appearance is not diagnostic except when candidiasis is present. 245) Following a supraglottic laryngectomy, the patient is taught how to use the supraglottic swallow to minimize the risk of aspiration. In teaching the patient about this technique, the nurse instructs the patient to A. perform Valsalva maneuver immediately after swallowing. B. breathe between each Valsalva maneuver and cough sequence. C. cough after swallowing to remove food from the top of the vocal cords. D. practice swallowing thin, watery fluids before attempting to swallow solid foods. C. cough after swallowing to remove food from the top of the vocal cords. A supraglottic laryngectomy involves removal of the epiglottis and false vocal cords, and the removal of the epiglottis allows food to enter the trachea. Supraglottic swallowing requires performance of the Valsalva maneuver before placing food in the mouth and swallowing. The patient then coughs to remove food from the top of the vocal cords, swallows again, and then breathes after the food has been removed from the vocal cords. 246) A patient is admitted to the hospital with fever, chills, a productive cough with rusty sputum, and pleuritic chest pain. Pneumococcal pneumonia is suspected. An appropriate nursing diagnosis for the patient based on the patient's manifestations is A. hyperthermia related to acute infectious process. B. chronic pain related to ineffective pain management. C. risk for injury related to disorientation and confusion. D. ineffective airway clearance related to retained secretions. A. hyperthermia related to acute infectious process. The patient with pneumococcal pneumonia is acutely ill with fever and the systemic manifestations of fever, such as chills, thirst, headache, and malaise. Interventions that monitor temperature and aid in lowering body temperature are appropriate. Ineffective airway clearance would be manifested by adventitious breath sounds and difficulty producing secretions. Disorientation and confusion are not noted in this patient and are not typical unless the patient is very hypoxemic. Pleuritic pain is an acute pain that is due to inflammation of the pleura. 247) The resurgence in TB resulting from the emergence of multidrug-resistant strains of Mycobacterium tuberculosis is primarily the result of A. a lack of effective means to diagnose TB. B. poor compliance with drug therapy in patients with TB. C. the increased population of immunosuppressed individuals with AIDS. D. indiscriminate use of antitubercular drugs in treatment of other infections. B. poor compliance with drug therapy in patients with TB. Drug-resistant strains of TB have developed because TB patients' compliance to drug therapy has been poor and there has been general decreased vigilance in monitoring and follow-up of TB treatment. Antitubercular drugs are almost exclusively used for TB infections. TB can be effectively diagnosed with sputum cultures. The incidence of TB is at epidemic proportions in patients with HIV, but this does not account for drug-resistant strains of TB. 248) The chronic inflammation of the bronchi characteristic of chronic obstructive pulmonary disease (COPD) results in A. collapse of small bronchioles on expiration. B. permanent, abnormal dilation of the bronchi. C. hyperplasia of mucus-secreting cells and bronchial edema. D. destruction of the elastic and muscular structures of the bronchial wall. C. hyperplasia of mucus-secreting cells and bronchial edema. Chronic bronchitis is characterized by chronic inflammation of the bronchial lining, with edema and increased mucus production. Collapse of small bronchioles on expiration is common in emphysema, and abnormal dilation of the bronchi because of destruction of the elastic and muscular structures is characteristic of bronchiectasis. 249) In teaching the patient with COPD about the need for physical exercise, the nurse informs the patient that A. all patients with COPD should be able to increase walking gradually up to 20 min/day. B. a bronchodilator inhaler should be used to relieve exercise-induced dyspnea immediately after exercise. C. shortness of breath is expected during exercise but should return to baseline within 5 minutes after the exercise. D. monitoring the heart rate before and after exercise is the best way to determine how much exercise can be tolerated. C. shortness of breath is expected during exercise but should return to baseline within 5 minutes after the exercise.Shortness of breath usually increases during exercise, but the activity is not being overdone if breathing returns to baseline within 5 minutes after stopping. Bronchodilators can be administered 10 minutes before exercise but should not be administered for at least 5 minutes after activity to allow recovery. Patients are encouraged to walk 15 to 20 minutes a day with gradual increases, but actual patterns will depend on patient tolerance. Dyspnea most frequently limits exercise and is a better indication of exercise tolerance than is heart rate in the patient with COPD. 250) 39. Select all that apply. Which of the following are clinical manifestations of tension pneumothorax? A. Midline trachea B. Severe hypertension C. Progressive cyanosis D. A loud bruit on affected side E. Asymmetrical chest wall movement F. Subcutaneous emphysema in the neck C,E, F The indicators of tension pneumothorax are asymmetrical chest wall movement, severe hypotension, subcutaneous emphysema in the neck and upper chest, and progressive cyanosis. [Show More]

Last updated: 1 year ago

Preview 1 out of 118 pages

Reviews( 0 )

$17.00

Add to cart

Instant download

Can't find what you want? Try our AI powered Search

OR

GET ASSIGNMENT HELP
156
0

Document information


Connected school, study & course


About the document


Uploaded On

Apr 29, 2021

Number of pages

118

Written in

Seller


seller-icon
A+ Solutions

Member since 3 years

164 Documents Sold


Additional information

This document has been written for:

Uploaded

Apr 29, 2021

Downloads

 0

Views

 156

Document Keyword Tags

Recommended For You


$17.00
What is Browsegrades

In Browsegrades, a student can earn by offering help to other student. Students can help other students with materials by upploading their notes and earn money.

We are here to help

We're available through e-mail, Twitter, Facebook, and live chat.
 FAQ
 Questions? Leave a message!

Follow us on
 Twitter

Copyright © Browsegrades · High quality services·